- Sat Apr 09, 2016 11:02 am
#22911
Complete Question Explanation
Justify the Conclusion. The correct answer choice is (A)
The conclusion in this stimulus is that insurance company regulations which deny coverage for lab tests decrease the quality of health care. This conclusion requires a couple assumptions. First, that the insurance restrictions are leading to reduced use of tests. (This seems like a plausible assumption according to ordinary economic reasoning, but it is not stated in the stimulus.) Second, that the lab tests actually improve the quality of health care.
The question stem asks us to "justify" the conclusion, meaning that a single correct answer choice will fill in all the missing assumptions and make the argument air-tight.
Answer choice (A): This is the correct answer choice. This answer choice states that the lab tests improve quality of medical care. This is the best answer choice, although it does not exactly "justify" the conclusion as the question stem asks. Note that the argument still has a missing assumption: it has failed to establish that the insurance restrictions are reducing use of the tests by patients. The LSAT test makers did not choose their words very carefully here.
Answer choice (B): The personal opinion of physicians regarding the insurance restrictions does not give us a clear implication about whether these lab tests improve quality of care. Perhaps physicians favor these tests because they truly provide value for patients \96 or perhaps physicians favor these tests because they are good revenue generators.
Answer choice (C): If some patients who might benefit from the tests have no health insurance whatsoever, then the insurance restrictions do not affect these patients. A reduction of quality of care is thus impossible for these patients (unless we make the roundabout argument that doctors lose revenue and thus the overall quality of their medical practice suffers). This answer choice tends to weaken the stimulus argument.
Answer choice (D): Even if "some" illnesses can be diagnosed without lab tests, this answer choice does not speak to the issue of whether lab tests improve quality of medical care. If anything, this answer choice weakens the stimulus by showing that lab tests are not quite so valuable as we might suppose.
Answer choice (E): This answer choice does not speak to the issues of (1) value added for patients by the lab tests, or (2) whether the insurance restrictions are reducing use of the lab tests.
Justify the Conclusion. The correct answer choice is (A)
The conclusion in this stimulus is that insurance company regulations which deny coverage for lab tests decrease the quality of health care. This conclusion requires a couple assumptions. First, that the insurance restrictions are leading to reduced use of tests. (This seems like a plausible assumption according to ordinary economic reasoning, but it is not stated in the stimulus.) Second, that the lab tests actually improve the quality of health care.
The question stem asks us to "justify" the conclusion, meaning that a single correct answer choice will fill in all the missing assumptions and make the argument air-tight.
Answer choice (A): This is the correct answer choice. This answer choice states that the lab tests improve quality of medical care. This is the best answer choice, although it does not exactly "justify" the conclusion as the question stem asks. Note that the argument still has a missing assumption: it has failed to establish that the insurance restrictions are reducing use of the tests by patients. The LSAT test makers did not choose their words very carefully here.
Answer choice (B): The personal opinion of physicians regarding the insurance restrictions does not give us a clear implication about whether these lab tests improve quality of care. Perhaps physicians favor these tests because they truly provide value for patients \96 or perhaps physicians favor these tests because they are good revenue generators.
Answer choice (C): If some patients who might benefit from the tests have no health insurance whatsoever, then the insurance restrictions do not affect these patients. A reduction of quality of care is thus impossible for these patients (unless we make the roundabout argument that doctors lose revenue and thus the overall quality of their medical practice suffers). This answer choice tends to weaken the stimulus argument.
Answer choice (D): Even if "some" illnesses can be diagnosed without lab tests, this answer choice does not speak to the issue of whether lab tests improve quality of medical care. If anything, this answer choice weakens the stimulus by showing that lab tests are not quite so valuable as we might suppose.
Answer choice (E): This answer choice does not speak to the issues of (1) value added for patients by the lab tests, or (2) whether the insurance restrictions are reducing use of the lab tests.